Finding the angular momentum using the inertia tensor/matrix

In summary, the problem involves finding the angular momentum of a thin ring with a radius r, constrained to rotate with constant angular velocity ω, and a linear mass density ρ(θ) = ρ0(2+sin2θ). The moment of inertia is calculated twice, using moments and products of inertia and by directly integrating L=∫dmr x v. However, the integral limits and variables used in the integrands may not be correct and should be reviewed.
  • #1
shanepitts
84
1

Homework Statement


A thin ring of radius r is constrained to rotate with constant angular velocity ω as shown in attached picture. Let the linear mass density of the ring be ρ(θ)=ρ0(2+sin2θ) where ρ0 is constant.

a) Find the angular momentum L of the ring about O, at the instant the ring is in the xy plane as shown. Answer this part twice: (i) by using the moments and products of inertia Iij, and (ii) by directly integrating L=∫dmr x v.
image.jpeg


Homework Equations


The moment of inertia tensor/ matrix.
L==ntransposeIn

The Attempt at a Solution


Not sure if I am starting this problem properly, attached below is my attempt.
image.jpeg


Knowing that I must plug these moments and products of India inside the tensor matrix.

Please help
 
Physics news on Phys.org
  • #2
When you calculated the moment of inertia Ixx, you treated the density of the ring, ρ, as a constant. According to the OP, ρ(θ) = ρo(2+sin2θ), where ρo is a constant. If you are going to calculate the MOI matrix for the ring, you must take this arbitrary density function into account. This extends even to calculating the mass of the ring.
 
  • Like
Likes shanepitts
  • #3
SteamKing said:
When you calculated the moment of inertia Ixx, you treated the density of the ring, ρ, as a constant. According to the OP, ρ(θ) = ρo(2+sin2θ), where ρo is a constant. If you are going to calculate the MOI matrix for the ring, you must take this arbitrary density function into account. This extends even to calculating the mass of the ring.

Thank you.

But are my integral limits correct considering it is a ring? Moreover, shall I integrate with respect to y, x, and/or θ, or just one variable?
 
  • #4
shanepitts said:
Thank you.

But are my integral limits correct considering it is a ring? Moreover, shall I integrate with respect to y, x, and/or θ, or just one variable?
I don't think so.

You seem to have integrands which use cartesian coordinates while the limits appear to be expressed in polar coordinates (what does x = 2πr mean? Isn't that the circumference of the ring?)
 
  • Like
Likes shanepitts
  • #5
SteamKing said:
I don't think so.

You seem to have integrands which use cartesian coordinates while the limits appear to be expressed in polar coordinates (what does x = 2πr mean? Isn't that the circumference of the ring?)

Thanks.
 

Related to Finding the angular momentum using the inertia tensor/matrix

1. What is angular momentum?

Angular momentum is a measure of an object's rotational motion, and it is defined as the product of an object's moment of inertia and its angular velocity.

2. What is the inertia tensor/matrix?

The inertia tensor/matrix is a mathematical representation of an object's moment of inertia, which is a measure of an object's resistance to changes in its rotational motion. It takes into account the object's mass distribution and the axis of rotation.

3. How is the inertia tensor/matrix used to calculate angular momentum?

The inertia tensor/matrix is used in the formula for angular momentum, which is L = Iω, where L is the angular momentum, I is the inertia tensor/matrix, and ω is the angular velocity. By plugging in the values for I and ω, the angular momentum can be calculated.

4. What is the significance of finding the angular momentum using the inertia tensor/matrix?

Using the inertia tensor/matrix to calculate angular momentum allows for a more accurate representation of an object's rotational motion, as it takes into account the object's mass distribution and the axis of rotation. This is especially important in situations where the object is not a simple geometric shape.

5. How is the inertia tensor/matrix determined for a complex object?

The inertia tensor/matrix can be determined for a complex object by dividing the object into smaller, simpler shapes and using the parallel axis theorem to find the moment of inertia for each shape. These individual moments of inertia can then be combined to calculate the inertia tensor/matrix for the entire object.

Similar threads

Replies
4
Views
1K
  • Advanced Physics Homework Help
Replies
1
Views
2K
  • Advanced Physics Homework Help
Replies
3
Views
1K
Replies
5
Views
1K
  • Introductory Physics Homework Help
2
Replies
45
Views
2K
  • Advanced Physics Homework Help
Replies
5
Views
1K
  • Quantum Physics
Replies
11
Views
1K
  • Introductory Physics Homework Help
Replies
10
Views
967
Replies
12
Views
3K
  • Advanced Physics Homework Help
Replies
3
Views
3K
Back
Top